21: Which of the following statements about the use of magnesium in cardiac arrest is most accurate? Magnesium is indicated for shock-refractory monomorphic VT. Magnesium is indicated for VF/pulseless VT associated with torsades de pointes. Magnesium is contraindicated for VT associated with a normal QT interval. Magnesium is indicated for VF refractory to shock and amiodarone or lidocaine. 22: A patient with ST-segment elevation MI has ongoing chest discomfort. Fibrinolytic therapy has been ordered. Heparin 4000 units IV bolus was administered, and a heparin infusion of 1000 units per hour is being administered. Aspirin was not taken by the patient because he had a history of gastritis treated 5 years ago. Your next action is to: Give aspirin 160 to 325 mg chewed immediately. Give 75 mg enteric-coated aspirin orally. Give 325 mg enteric-coated aspirin rectally. Substitute clopidogrel 300 mg loading dose. 23: A patient has sinus bradycardia with a heart rate of 36/min. Atropine has been administered to a total of 3 mg. A transcutaneous pacemaker has failed to capture. The patient is confused, and her blood pressure is 110/60 mm Hg. Which of the following is now indicated? A. Give additional 1 mg atropine. B. Start dopamine 10 to 20 mcg/kg per minute. C. Give normal saline bolus 250 mL to 500 mL. D. Start epinephrine 2 to 10 mcg/min. 24: A 62-year-old man suddenly experienced difficulty speaking and left-side weakness. He was brought to the emergency department. He meets initial criteria for fibrinolytic therapy, and a CT scan of the brain is ordered. What are the guidelines for antiplatelet and fibrinolytic therapy> A. Do not give aspirin for at least 24 hours if rtPA is administered. B. Give aspirin 160 mg and clopidogrel 75 mg orally. C. Administer heparin if CT scan is negative for hemorrhage. D. Administer aspirin 160 to 325 mg chewed immediately. 25: A patient with possible ST-segment elevation MI has ongoing chest discomfort. Which of the following would be a contraindication to the administration of nitrates? A. Heart rate 90/min. C. Blood pressure greater than 180 mm Hg. B. Left ventricular infarct with bilateral rales. D Use of a phosphodiesterase inhibitor within 12 hours 26: A patient is in cardiac arrest. Ventricular fibrillation has been refractory to a second shock. Of the following, which drug and dose should be administered first by the IV/IO route? A. Epinephrine 1 mg B. Vasopressin 20 units C. Sodium bicarbonate 50 mEq D. Atropine 1 mg 27: A 35-year-old woman has palpitations, light-headedness, and a stable tachycardia. The monitor shows a regular narrowcomplex QRS at a rate of 180/min. Vagal maneuvers have not been effective in terminating the rhythm. An IV has been established. What drug should be administered IV? A. Lidocaine 1mg/kg B. Adenosine 6 mg C. Epinephrine 2 to 10 mcg/kg per minute D. Atropine 0.5 mg 28: A patient with sinus bradycardia and heart rate of 42/min has diaphoresis and a blood pressure of 80/60 mm Hg. What is the initial dose of atropine? A. 0.1mg B. 3 mg C. 1 mg D. 0.5 mg 29: A patient is in refractory ventricular fibrillation and has received multiple appropriate defibrillation shocks, epinephrine 1 mg IV twice, and an initial dose of 300 mg amiodarone IV. The patient is intubated. A second dose of amiodarone is now called for. The recommended second dose of amiodarone is: A. An endotracheal dose of 2 to 4 mg/kg. B. 300 mg IV push. C. 1 mg/kg IV push. This study source was downloaded by 100000828331062 from CourseHero.com on 10-16-2022 23:41:37 GMT -05:00 https://www.coursehero.com/file/49838176/219953798-ACLS-Post-Testdocx/ D. An infusion of 1 to 2 mg/min. E. 150 mg IV push. 30: A patient with a possible acute coronary syndrome has ongoing chest discomfort unresponsive to 3 sublingual nitroglycerin tablets. There are no contraindications, and 4 mg of morphine sulfate was administered. Shortly afterward, blood pressure falls to 88/60 mm Hg, and the patient has increased chest discomfort. You should: A. Give normal saline 250 mL to 500 mL fluid bolus. B. Give an additional 2 mg of morphine sulfate. C. Give sublingual nitroglycerin 0.4 mg. D. Start dopamine at 2 mcg/kg per minute and titrate to a systolic blood pressure reading of 100 mm Hg. 31: A patient has a rapid irregular wide-complex tachycardia. The ventricular rate is 138/min. He is asymptomatic, with a blood pressure of 110/70 mm Hg. He has a history of angina. Which of the following actions is recommended? A. Giving adenosine 6 mg IV bolus. B. Seeking expert consultation. C. Giving lidocaine 1 to 1.5 mg IV bolus. D. Immediate synchronized cardioversion. 32: You arrive on the scene with the code team. High-quality CPR is in progress. An AED has previously advised “no shock indicated.” A rhythm check now finds asystole. After resuming high-quality compressions, your next action is to: A. Gain IV or IO access. B. Place an esophageal-tracheal tube or laryngeal mask airway. C. Attempt endotracheal intubation with minimal interruptions in CPR. D. Call for a pulse check. 33: A patient is in pulseless ventricular tachycardia. Two shocks and 1 dose of epinephrine have been given. Which is the next drug/dose to anticipate to administer? A. Amiodarone 300 mg B. Amiodarone 150 mg C. Vasopressin 40 units D. Epinephrine 3 mg E. Lidocaine 0.5 mg/kg 34: Your patient has been intubated. IV/IO access is not available. Which combination of drugs can be administered by the endotracheal route? A. Vasopressin, amiodarone, lidocaine B. Amiodarone, lidocaine, epinephrine C. Epinephrine, vasopressin, amiodarone D. X Lidocaine, epinephrine, vasopressin 35: A patient is in cardiac arrest. Ventricular fibrillation has been refractory to an initial shock. What is the recommended route for drug administration during CPR? A. Femoral vein B. X IV or IO C. Central line D. Endotracheal E. External jugular vein 36: A patient is in refractory ventricular fibrillation. High-quality CPR is in progress, and shocks have been given. One does of epinephrine was given after the second shock. An antiarrhythmic drug was given immediately after the third shock. What drug should the team leader request to be prepared for administration next? A. Escalating dose of epinephrine 3 mg. C. Repeat the antiarrhythmic drug B. Second dose of epinephrine 1 mg D. Sodium bicarbonate 50 mEq Question 37: A 57-year-old woman has palpitations, chest discomfort, and tachycardia. The monitor shows a regular widecomplex ORS at a rate of 180/min. She becomes diaphoretic, and her blood pressure is 80/60 mm Hg. The next action is to: Give amiodarone 300 mg IV push. 

No comments found.
Login to post a comment
This item has not received any review yet.
Login to review this item
No Questions / Answers added yet.
Price $10.00
Add To Cart

Buy Now
Category exam bundles
Comments 0
Rating
Sales 0

Buy Our Plan

We have

The latest updated Study Material Bundle with 100% Satisfaction guarantee

Visit Now
{{ userMessage }}
Processing